Formatierung von mehreren Gleichungen untereinander

Schriftbild, Absätze und Auflistungen einstellen


MAB
Forum-Newbie
Forum-Newbie
Beiträge: 5
Registriert: Di 18. Apr 2017, 16:00

Formatierung von mehreren Gleichungen untereinander

Beitrag von MAB »

Hallo,

ich möchte innerhalb einer align-Umgebung (oder alignat) mehrere untereinander stehende Gleichungen, wovon die ersten vier Gleichungen deutlich kürzer (weniger Terme) sind als die letzten Gleichungen.
Für das Minimalbeispiel habe ich die Terme mal abgekürzt durch Platzhalter, aber die Problematik dürfte dennoch deutlich werden.
% Das ist mein Minimalbeispiel
\documentclass{article}
\usepackage{amsmath}

\begin{document}

\noindent Wie es schematisch aussieht!

\begin{alignat}{2}
	A &= \qquad A_1 \\
%
	B &= \qquad B_1 \\
%
	C &= \qquad C_1 \\
%
	D &= \qquad D_1 \\
%
	E &= E_1 = &&E_2	+ \nonumber \\ {}&{}&&+ E_3 \\
	  &= E_4 = &&E_5	+ \nonumber \\ {}&{}&&+ E_6 \\       
%      
	F &= F_1 = &&F_2	+ \nonumber \\ {}&{}&&+ F_3 \\
%
	G &= G_1 = &&G_2	+ \nonumber \\ {}&{}&&+ G_3 
\end{alignat}

\noindent Wie es schematisch aussehen soll!

\begin{alignat}{2}
	A &= A_1 \\
%
	B &= B_1 \\
%
	C &= C_1 \\
%
	D &= D_1 \\
%
	E &= E_1 = &&E_2	+ \nonumber \\ {}&{}&&+ E_3 \\
	  &= E_4 = &&E_5	+ \nonumber \\ {}&{}&&+ E_6 \\       
%      
	F &= F_1 = &&F_2	+ \nonumber \\ {}&{}&&+ F_3 \\
%
	G &= G_1 = &&G_2	+ \nonumber \\ {}&{}&&+ G_3 
\end{alignat}

\end{document}

% Ende des Minimalbeispiels
Was mich aktuell stört und was ich beheben möchte: die letzten drei Gleichungen bestehen aus zwei jeweils Gleichungen, d.h. zweimal ein Gleichheitszeichen. Ich möchte, dass die Gleichheitszeichen untereinander stehen, aber ich möchte auch, dass bei den letzten drei Gleichungen nicht so eine große Lücke zwischen der rechten Seite der Gleichung und dem zweiten Gleichheitszeichen erscheint.

Ich hoffe, dass man mir weiterhelfen kann.
Viele Grüße
Zuletzt geändert von MAB am Mi 22. Nov 2017, 14:48, insgesamt 2-mal geändert.

Gast

Beitrag von Gast »

Dein Beispiel ist unvollständig.

MAB
Forum-Newbie
Forum-Newbie
Beiträge: 5
Registriert: Di 18. Apr 2017, 16:00

Beitrag von MAB »

Aha und was fehlt?

Bartman
Forum-Meister
Forum-Meister
Beiträge: 2456
Registriert: Do 16. Jul 2009, 21:41
Wohnort: Hessische Provinz

Beitrag von Bartman »

Gemäß Deiner Beschreibung im Eröffnungsbeitrag bist Du in der Lage, eine PDF-Datei zu erzeugen. Geht das auch mit Deinem Beispiel?

Stamm-

Körper des Dokuments fehlt

Beitrag von Stamm- »

MAB hat geschrieben:Aha und was fehlt?
Diese Frage impliziert ja schon, dass Du das Beispiel hier reingestellt hast, ohne es vorher zu testen und damit auf Lauffähigkeit zu prüfen. Und um die Frage zu beantworten: Es fehlt die Umgebung 'document'.

MAB
Forum-Newbie
Forum-Newbie
Beiträge: 5
Registriert: Di 18. Apr 2017, 16:00

Beitrag von MAB »

Mimimi, ich habe mal ein PDF beigefügt, das illustrieren soll, wie die Gleichungen in etwa schematisch dargestellt werden und wie ich es gerne hätte. Den Originalcode mit den entsprechenden Gleichungen werde ich aus Datenschutzgründen hier nicht posten, aber sicherlich verfügt man über genügend abstraktes Denken, dass man sich hinter den Platzhaltern beliebige mathematische Terme vorstellen kann.
Dateianhänge
Minimalbeispiel.pdf
(17.18 KiB) 302-mal heruntergeladen

markusv
Forum-Meister
Forum-Meister
Beiträge: 947
Registriert: Do 3. Sep 2015, 17:20
Wohnort: Leipzig

Beitrag von markusv »

MAB hat geschrieben:Mimimi
Du weißt, dass hier niemand gezwungen ist, Dir zu helfen und alles auf persönlichem Einsatz und Opferung seiner Freizeit basiert? Glaubst du, jemand verschwendet seine kostbare Zeit damit, Dir zu helfen, wenn du, der das Problem hat, nicht einmal selbst soviel Zeit aufbringen will, um einen vernünftigen Post zu erstellen?!?

Aufgrund deiner eigenen Anpassungen sehe ich das Thema als beantwortet?!? Wenn ja, dann ändere bitte den Status entsprechend.
Wäre Microsoft Word für das Schreiben von Büchern entwickelt worden,
würde es Microsoft Book heißen.

Unkomplizierte und schnelle LaTeX-Hilfe, bspw. Erstellung von Vorlagen und Bewerbungen:
Help-LaTeX@web.de

MAB
Forum-Newbie
Forum-Newbie
Beiträge: 5
Registriert: Di 18. Apr 2017, 16:00

Beitrag von MAB »

Ganz ehrlich, man hätte auch eingangs freundlich kommunizieren können, was da noch fehlt. Dass da noch \begin{document} bzw. \end{document} zunächst noch fehlte, war ein reiner Flüchtigkeitsfehler und da finde ich es einfach nur etwas albern, wenn man dann in einem LaTeX-Forum so tut, als ob man mit dem Quellcode ansonsten gar nichts anfangen könne. Allein die Tatsache, dass es dann vier Beiträge benötigte, bis deutlich wurde, dass obiger Code noch hinzugefügt werden musste, ist einfach unnötig.

Mein LaTeX-Problem ist nach wie vor ungelöst und wenn mir dabei jemand helfen mag, freue ich mich, ansonsten wird sich sicher anderweitig eine Lösung finden.
Ich kann gerne das Beispiel nochmal etwas konkreter bearbeiten, aber man muss auch Verständnis dafür haben, dass ich nicht den Orginalcode einfügen kann und darf, da die Gleichungen aktuell nicht veröffentlicht werden dürfen.

Ich kann zunächst nochmal in Worten beschreiben, was das Problem ist.
Die ersten vier Gleichungen bestehen aus lediglich je einer Gleichung (also ein Gleichheitszeichen). Die letzten drei Gleichungen besitzen jedoch noch einen mittlere Gleichung, also jeweils zwei Gleichheitszeichen.
Momentan kriege ich es noch nicht richtig gelöst, mittels "Ausrichtungsankern" (also dem &) die einzelnen Gleichungen bzw. Terme so auszurichten, dass keine "unnatürlich" großen Lücken vor oder hinter den Gleichheitszeichen auftreten. Die Reihenfolge der Gleichungen kann übrigens nicht verändert werden.
Ich hoffe, dass einigermaßen deutlich wird, was das Problem ist, was ich mit störenden Lücken meine und dass ich diese gerne beheben wollen würde.

markusv
Forum-Meister
Forum-Meister
Beiträge: 947
Registriert: Do 3. Sep 2015, 17:20
Wohnort: Leipzig

Beitrag von markusv »

Oder anders: man kann nach dem Posten sein eigenen Code überprüfen und feststellen, dass der Code nicht lauffähig ist. Die fehlende Umgebung wird sogar als Fehlermeldung angezeigt. Benötigte Posts: 0.

Aber naja, Schluss jetzt. Meine Meinung dazu habe ich denke ich kundgetan.

Leider verstehe ich dein Problem nicht ganz. Ist es nicht absolut logisch, dass, wenn du alles an den Gleichheitszeichen ausrichten möchtest und die Terme unterschiedliche Längen haben, bei den kürzeren Termen Leerräume entstehen?

Für mein Verständnis könntest du dein Beispiel mit geschwärzten Balken aufbauen und kennzeichnen bzw. beschreiben, wo die unerwünschten Leerräume auftreten.

Alternativ könntest du die Umgebung wechseln und bspw. array nutzen. Hier kannst du sämtliche Tabellenbefehle wie bspw. \multicolumn nutzen. Vielleicht hilft das ja.
\documentclass{article}
\usepackage{amsmath}
\begin{document}
\begin{alignat}{2}
   A &= A_1 \\
%
   B &= B_1 \\
%
   C &= C_1 \\
%
   D &= D_1 \\
%
   E &= \rule{50pt}{10pt}  && = E_2 + \nonumber \\
     &= \rule{25pt}{10pt}  && = E_5\\ %ausgerichtet am = darüber
     &= E_4 = \rule{25pt}{10pt} % ohne Platz zwischen Gleichung und =
\end{alignat}
\[
\begin{array}{rlll}
   A &= A_1 \\
   B &= B_1 \\
   C &= C_1 \\
   D &= D_1 \\
   E &= \rule{25pt}{10pt} & = E_2\\
     &\multicolumn{2}{l}{= \rule{75pt}{10pt} = E_5}\\%multicolumn
     &= E_4 & = \rule{25pt}{10pt}\\%ausgerichtet am = der vorletzten Gleichung
\end{array}
\]
\end{document}
Wäre Microsoft Word für das Schreiben von Büchern entwickelt worden,
würde es Microsoft Book heißen.

Unkomplizierte und schnelle LaTeX-Hilfe, bspw. Erstellung von Vorlagen und Bewerbungen:
Help-LaTeX@web.de

esdd
Forum-Meister
Forum-Meister
Beiträge: 2561
Registriert: So 7. Feb 2010, 16:36

Beitrag von esdd »

Ich verstehe das zuerreichende Ziel auch nicht. Deshalb hier mal nur noch eine Variante mit multlined aus dem mathtools Paket, bei dem die zweiten Gleichheitszeichen nicht aneinander ausgerichtet werden:
\documentclass{article}
\usepackage{mathtools}% lädt auch amsmath
\begin{document}
\begin{align}
   A & = A_1 \\
   B & = B_1 \\
   C & = C_1 \\
   D & = D_1 \\
   E & = E_1 = \begin{multlined}[t] E_2 + {} \\ + E_3 \end{multlined} \\
     & = E_4 = \begin{multlined}[t] E_5 + {} \\ + E_6 \end{multlined} \\       
   F & = F_1 = \begin{multlined}[t] F_2 + {} \\ + F_3 \end{multlined} \\
   G & = G_1 = \begin{multlined}[t] G_2 + {} \\ + G_3 \end{multlined}
\end{align}
\end{document}
Zuletzt geändert von esdd am Do 30. Nov 2017, 14:41, insgesamt 1-mal geändert.

Antworten